- PowerScore Staff
- Posts: 5972
- Joined: Mar 25, 2011
- Sat May 28, 2016 2:33 pm
#87411
Complete Question Explanation
(The complete setup for this game can be found here: lsat/viewtopic.php?f=149&t=16450)
The correct answer choice is (E)
This is the other Justify question in this game. The question stem asks you to supply a condition that forces L to have been begun in 602. From question #3, we know that placing S into 604 achieves that objective, but S in 604 is not one of the answers. However, S in 603 is one of the answers, and given that similarity, exploring the answer choice with S in 603 first is advisable.
When S is begun in 603, G and M must be begun in 601. As L cannot be in 604 or 605, L must therefore be in 602.
Accordingly, answer choice (E) is correct.
Note that in both question #3 and question #5, the correct answer addresses S, a random. While having a random appear as the correct answer in two Justify questions may at first be surprising, there is logic behind this occurrence. In games where only one random exists, sometimes placing a restriction on the one unrestricted variable creates a situation where the other variables are then forced into place. That is the situation here, and the principle is that adding a restriction to the one unrestricted variable can sometimes cause the remaining restricted variables to fall into line.
(The complete setup for this game can be found here: lsat/viewtopic.php?f=149&t=16450)
The correct answer choice is (E)
This is the other Justify question in this game. The question stem asks you to supply a condition that forces L to have been begun in 602. From question #3, we know that placing S into 604 achieves that objective, but S in 604 is not one of the answers. However, S in 603 is one of the answers, and given that similarity, exploring the answer choice with S in 603 first is advisable.
When S is begun in 603, G and M must be begun in 601. As L cannot be in 604 or 605, L must therefore be in 602.
Accordingly, answer choice (E) is correct.
Note that in both question #3 and question #5, the correct answer addresses S, a random. While having a random appear as the correct answer in two Justify questions may at first be surprising, there is logic behind this occurrence. In games where only one random exists, sometimes placing a restriction on the one unrestricted variable creates a situation where the other variables are then forced into place. That is the situation here, and the principle is that adding a restriction to the one unrestricted variable can sometimes cause the remaining restricted variables to fall into line.
Dave Killoran
PowerScore Test Preparation
Follow me on X/Twitter at http://twitter.com/DaveKilloran
My LSAT Articles: http://blog.powerscore.com/lsat/author/dave-killoran
PowerScore Podcast: http://www.powerscore.com/lsat/podcast/
PowerScore Test Preparation
Follow me on X/Twitter at http://twitter.com/DaveKilloran
My LSAT Articles: http://blog.powerscore.com/lsat/author/dave-killoran
PowerScore Podcast: http://www.powerscore.com/lsat/podcast/